Sp.metrice4

Embed Size (px)

Citation preview

  • 8/13/2019 Sp.metrice4

    1/3

    The University of Sydney

    MATH 3901

    Metric Spaces 2004

    Tutorial 4

    PROBLEM SET 4

    1. If a sequence (xn) in a metric space Xis convergent and has limitx, show thatevery subsequence (xnk) of (xn) is convergent and has the same limit x.

    Solution.

    Let >0. Since xn x as n , there is N such that n > N implies

    d(xn, x)< .

    Thus, ifnk > N, thend(xnk , x)< . Hence (xnk) x as k .

    2. Let M be the metric subspace consisting of all sequences x= (xk) withat most finitely many nonzero terms, where d is given by

    d(x, y) = supkN

    |xk yk|.

    Show thatMis not closed. [Hint: Try to produce a sequence in Mwhich doesnot converge to a point in M.]

    Solution.

    For n= 1, 2, . . . , let

    x(n) =

    1, 1

    2, . . . ,

    1

    n, 0, 0, . . .

    .

    Then (x(n)) is a sequence in M. Let x be defined by

    x=

    1, 1

    2, . . . ,

    1

    n, . . .

    .

    Thenx butx / M. Moreover, for n > N, we see that

    d(x(n), x) = sup 1

    n+ 1,

    1

    n+ 2, . . .

    =

    1

    n+ 10, we choose N >1/ so that for any n > N,

    d(x(n), x)< 1

    N+ 10, we

    must show that there is a >0 such that ifx satisfies |x x0| < , then

    |f(x) f(x0)| < .

    Now, for such , we consider /3. Since fn f, there is N such that for alln N, d(fn, f)< /3; that is, for all x [a, b], we have for all n N that

    |fn(x) f(x)| < /3.

    Since fN is continuous at x0, for such /3, there exists > 0 such that ifx

    satisfies|x x0| < , then

    |fN(x) fN(x0)| < /3 .

    Hence ifx satisfies |x x0| < , then

    |f(x) f(x0)| < |f(x) fN(x)| + |fN(x) fN(x0)| + |fN(x0) f(x0)|

    <

    3+

    3+

    3= .

    Hence fis continuous on [a, b].

  • 8/13/2019 Sp.metrice4

    3/3

    3

    5. Consider the metric spaceC[a, b], withd defined by

    d(f, g) = supx[a,b]

    |f(x) g(x)|.

    If (fn) is a sequence in C[a, b] converges uniformly to a function f on [a, b],

    prove that ba

    fn(x) dx

    ba

    f(x) dx as n .

    Solution.

    We note that for all x [a, b],

    |fn(x) f(x)| d(fn, f) .

    Then we have

    ba

    fn(x) dx

    ba

    f(x) dx

    ba

    |fn(x) f(x)| dx

    ba

    d(fn, f) dx

    (b a)d(fnf) 0,

    as n . Hence

    b

    a

    fn(x) dx

    b

    a

    f(x) dx as n .